LSAT and Law School Admissions Forum

Get expert LSAT preparation and law school admissions advice from PowerScore Test Preparation.

 Administrator
PowerScore Staff
  • PowerScore Staff
  • Posts: 8916
  • Joined: Feb 02, 2011
|
#43382
Please post your questions below! Thank you!
 lathlee
  • Posts: 652
  • Joined: Apr 01, 2016
|
#46171
I don't get why A) is the correct answer: considering the argument and the conclusion are about handheld vacuum cleaner sufficient and a) doesn't mention any of handheld vacuum cleaner capabilities or should be sufficient enough purchase.
 chian9010
  • Posts: 81
  • Joined: Jun 08, 2018
|
#57285
The question is a justification assumption which means that premise + assumption = conclusion

The conclusion is: An inexpensive handheld vacuum cleaner is likely to be sufficient for uncarpeted floors.
The premise: Most (handheld vacuum cleaners) are easy to use and will likely satisfy all your vacuuming needs on wood and tile floors.

=================
The mechanical strategy is to get rid of overlapped words in between premise and conclusion. And the correct answer choice should have the remaining (non-overlapped) key words - wood, tile, uncarpeted

Is this the correct strategy?
 James Finch
PowerScore Staff
  • PowerScore Staff
  • Posts: 943
  • Joined: Sep 06, 2017
|
#57697
Hi Chian,

This is a Strengthen question, which we can tell from the question stem ("most strongly support"), and the conclusion is the conditional statement in the first sentence. So here, while it won't be certain, the correct answer choice will act as an added premise making the conclusion more likely to be true (generally with hedge words like "most" or "likely"). The key here, as you note, is the logical gap between the "uncarpeted" floors in the conditional statement and the "wood or tile" mentioned in the premise. It's not as simple as repetition of words, as some words are synonymous, but rather the meaning. Here, wood and tile floors are subgroups of uncarpeted floors, but not necessarily the only ones. The clear path to strengthening this conclusion is to make the only options for uncarpeted floors be wood or tile.

Answer choice (A) does this, and is the correct answer.
 Shaela L. Hayes
  • Posts: 10
  • Joined: Jun 01, 2021
|
#88101
PowerScore,

Can you please explain why answer choice (D) is incorrect? I wrongly chose it twice for two different reasons (*cringe*) after narrowing the answers down to (A) and (D). First, I thought this was a justify question because of the combination of "assumed" and "supported" but I remember now that "most" automatically disqualifies the question from being justify because justify answers prove the conclusion 100%. The second time around I was attracted to the similar conditional reasoning in (D) and put off by the strong degree of (A). None of these reasons obviously follow any particular technique that can (or should) be applied in similar situations. Also, I don't see how any strengthen question type strategy taught in my course could have helped me correctly choose (A) over (D).

Thanks!
User avatar
 Poonam Agrawal
PowerScore Staff
  • PowerScore Staff
  • Posts: 71
  • Joined: Apr 23, 2021
|
#88134
Hi Shaela,

The biggest thing that jumps out to me about answer choice (D) is that it focuses on household cleaning needs, whereas the stimulus focuses on vacuuming needs (it is already implied in the stimulus that the consumer is in need of a vacuum). Hence, answer choice (D) doesn't particularly add any valuable information to what we already know from the stimulus.

On the other hand, the specificity of the language in answer choice (A) actually makes it a good answer here. The stimulus says that a handheld vaccum cleaner will probably satisfy all of your needs for tile and wood floors. What if most consumers have concrete floors? In that case, a handheld vacuum wouldn't be the best option for most people. Answer choice (A) removes this uncertainty by saying that most consumers do, in fact, have either carpet, wood, or tile floors. Therefore, if someone does have vacuuming needs for uncarpeted floors, it is likely that they will have wood/tile floors, which a handheld vacuum cleaner is good for. Thus, answer choice (A) strengthens the argument by removing a source of uncertainty about consumers' flooring types.

Hope that helps!
 Katherinthesky
  • Posts: 36
  • Joined: Feb 07, 2020
|
#90148
Hello!

Could you classify (C) as a Shell Game answer?

Thanks in advance!
 Rachael Wilkenfeld
PowerScore Staff
  • PowerScore Staff
  • Posts: 1358
  • Joined: Dec 15, 2011
|
#90273
Hi Katherinthesky.

Personally, I don't get too hung up in labeling incorrect answer choices. But you could call this a shell game in that it gives you similar terms that you see in the stimulus, but not in a way that actually addresses what we need the answer choice to do (here, strengthen). Answer choice (C) starts to fail when we realize it actually doesn't address the conclusion at all. We want to know that the vacuum is sufficient for the vacuuming needs. Our answer choice just describes it as sufficient only for cleaning small areas of uncarpeted floors.

Hope that helps!
 dimi.wassef@yahoo.com
  • Posts: 34
  • Joined: Aug 26, 2021
|
#92921
Hi,

I do not feel that the strength of the conclusion in this argument (likely sufficient) requires an assumption as strong as answer choice A. Can someone explain why E is wrong?
 Robert Carroll
PowerScore Staff
  • PowerScore Staff
  • Posts: 1787
  • Joined: Dec 06, 2013
|
#93010
dimi.wassef,

This is not an Assumption question, so no answer can be rejected because it's too strong. This is simply a Strengthen question, as shown by the phrase "most strongly supported".

Answer choice (E) does nothing to show that an inexpensive handheld vacuum cleaner is adequate. More versatile vacuums have a greater chance of being more expensive. That would possibly help show that a more versatile vacuum is more expensive, if that were our conclusion, or that an inexpensive vacuum is not versatile, if that were our conclusion. But neither of those is our conclusion. We're looking for a certain vacuum to be adequate for certain people. Answer choice (E) does nothing for that.

Robert Carroll

Get the most out of your LSAT Prep Plus subscription.

Analyze and track your performance with our Testing and Analytics Package.